Please answer max 10 min, I have a test.
Given a room is 12 feet by 10 feet and we have a circular rug with a radius of 3 feet, calculate the amount of floor left uncovered by the circular area rug in a rectangular living room

Answers

Answer 1

Step-by-step explanation:

floor = 12*10= 120 sq ft

rug = r² = 9 Пr2

120-9π = 91.74 sq ft


Related Questions

Sidney bought 18 toy cars. The cars came in p packages. Write an expression that shows
how many toy cars were in each package.

Answers

Answer:

[tex]\frac{18}{p}[/tex]

Step-by-step explanation:

A battery with a voltage of 3 V provides a voltage of 0.56 C to some circuits. Calculate the electrical power provided by this battery

Answers

Answer:

The power is 1.68 W.

Step-by-step explanation:

Voltage, V = 3 V

charge, q = 0.56 C

time, t = 1 s

Let the power is P.

P  = V q /t

P = 3 x 0.56 / 1 = 1.68 W

Given: Angle T S R and Angle Q R S are right angles; Angle T Is-congruent-to Angle Q Prove: Triangle T S R Is-congruent-to Triangle Q R S Triangles T S R and Q R S share side S R. Angles T S R and S R Q are right angles. Angles S T R and S Q R are congruent. Step 1: We know that Angle T S R Is-congruent-to Angle Q R S because all right angles are congruent. Step 2: We know that Angle T Is-congruent-to Angle Q because it is given. Step 3: We know that Line segment S R is-congruent-to line segment R S because of the reflexive property. Step 4: Triangle T S R Is-congruent-to Triangle Q R S because of the ASA congruence theorem. of the AAS congruence theorem. of the third angle theorem. all right triangles are congruent.

Answers

Answer:

Its b on edge

Step-by-step explanation:

yes

The Triangle TSR ≅ QRS by AAS congruence, the correct option is B.

What is a Triangle?

A triangle is a polygon with three sides, vertices and three angles.

The triangle TSR and QRS are right-angle triangles, in a right angle triangle one of the angle is equal to 90 degree,

'These triangles have to be proved congruent,

They both have the right angle.

The measure of Angle T = Angle Q,

They both have a common side,

So, they both are congruent by AAS congruence theorem.

To know more about Triangle

https://brainly.com/question/2773823

#SPJ5

Find the circumference of the circle. Round your answer to the nearest
hundredth.

Answers

Answer:

[tex]circumference = 25.13 \ cm[/tex]

Step-by-step explanation:

[tex]circumference = 2 \pi r = 2 \pi 4 = 8 \pi =25.13 \ cm[/tex]

Extend the sequence and then complete the
statements.
А
A (blank) is an ordered list of numbers that can
form a pattern.
А (blank)
is an element in a sequence.
You can (blank)
a sequence by finding and writing
more terms.
The next term in the sequence is A = (blank)

Answers

Answer:

l

Step-by-step explanation:

A sequence is an ordered list of numbers.....

A term is an element in a sequence.

You can expand a sequence by finding and writing....

The next term in the sequence is A = (general term)

A 2km long stretch of road needs to be resurfaced. The width of the road is 8.3m. Model this stretch of road as a rectangle.Work out the area of road that needs to be resurfaced in metre squared. Resurfacing this road costs £30 per square metre What is the total cost to resurface this road?

Answers

1 km = 1000 meters

2 km = 2 x 1000 = 2,000 meters

Area = length x width

Area = 2,000 x 8.3 = 16,600 square meters

Cost:

Multiply total area by cost per square meter:

16,600 x 30 = £498,000

Which equation represents a line which is parallel to the line
X + 6y = -24?

Answers

Answer:

y = -(1/6)x - 6

Step-by-step explanation:

slope intercept form is

y = mx + b

m is the slope

b is the y-intercept

---------------------------

x + 6y = -24

Subtract x from both sides

6y = -x  - 24

Divide both sides by 6

y = -(1/6)x - 4

slope = -1/6

-----------------------------

Parallel lines have the same slope

therefor the parallel line is

y = -(1/6)x - 6

A dress that costs $66.99 is on sale for 1/5 off. What is the sale price of the dress? Round your answer to the nearest cent.

Answers

Answer:

Step-by-step explanation:

The formula to figure this out is

[tex]C=66.99(1-.2)[/tex] where .2 is 1/5 in decimal form. What this means is that you may be saving 1/5 but you're still spending 4/5.

C = 66.99(.8) so

Cost = $53.59

the sale price of the dress is $53.59. you saved $13.40

A city has a population of 300,000 people. Suppose that each year the population grows by 4.5% . What will the population be after 8 years?

Answers

Answer:

population=300,000

population size per year=4.5%

population size for 8 years=8 × 4.5%

=0.36

=108000

Will mark brainliest

Answers

Answer:

the answer would be 270

Step-by-step explanation:

i can explain later:

since x = 5, you plug it in f = 4x +7, and you get f = 27 and since 1/2x is in the denominator, it will flip over and become 2x. 2x is 10, so you multiply f by g and get 270 which is if f/g(5) is the question.

Which is a correct first step in solving 5 – 2x < 8x – 3?

5 < 6x – 3
3x < 8x – 3
5 < 10x – 3
2 – 2x < 8

Answers

Answer:

5 < 10x - 3

Step-by-step explanation:

5 - 2x < 8x - 3

Add '2x' to both the sides

5 - 2x + 2x < 8x + 2x - 3

5 < 10x - 3

Twice a number is 16 greater than two times the opposite of the number. What is the number

Answers

Answer:

4

Step-by-step explanation:

Let's say our number is n.

First, we have two times a number. This can be represented by 2 times our number, n, or 2*n

Next, we have that is 16 greater than ... It is hard to conceptualize what 2*n is 16 greater than, so we can move on and save this for later.

Furthermore, we have two times the opposite of the number. Opposite represents the opposite side of a number line, e.g. if the number was 5, the opposite would be -5. This can be represented by multiplying our number n by -1, or -1 * n = -n. We then take two times of this opposite, or 2 * -1 * n = -2 * n.

So far, we have 2*n is 16 greater than -2 * n. We can then say that if a first number is 16 greater than a second number, we must add 16 to the second number to get the first number. Applying that here, we can say that 2*n = -2*n + 16

We then have

2*n = -2*n + 16

We then want to separate the n from the real number(s) (16 in this case). To do that, we can add 2*n to both sides, making our equation

2*n+2*n = 16

4*n = 16

Then, we can finally divide both sides by 4 to get

n = 4

what function is shown in the table

Answers

I think it’s a linear

What's the area of the shaded region?

Answers

Answer:

99cm²

Step-by-step explanation:

Area of shaded region=(area of outer rectangle)-(area of inner triangle)

Area of shaded region=(L×W)-(bh/2)

=(12×10)-((6×7)/2)

=120-(42/2)

=120-21

=99cm²

The area of the shaded region is 99cm2

jessica's piggy bank has 100 coins, including pennies, nickels, dimes and quarters/ together they are worth $8.36. there are nine more dimes than pennies and five times as many nickels as pennies. How many of each kind of coin does jessica have in her piggy bank?

Answers

Answer:

the bank contains 30 nickels and 70 dimes.

Step-by-step explanation:

Which ordered pair is a solution of the equation! Pic below! PLEASE HELP

Answers

Answer:

C

Step-by-step explanation:

[tex]5x+y=-23[/tex]

Graph it and see which plots are on the line:

-3, -4 is not.

-5, -3 is not.

-4, -3 is.

-3, -5 is not.

         

PLEASE HELP!!! I have 7 minutes to awnser this

Answers

Answer: It's A

Step-by-step explanation:

The correct answer is A

because the equation is in slope-intercept form “y=mx+b” and b is the y-intercept in the equation so it would be $75 which is the amount he was given

(a²+b²)= hihihihihihihihi

Answers

Answer:

? This isn't a problem

Step-by-step explanation:

Consider the sequence 2/4, 3/5, 4/6, 5/7,... Which statement describes the sequence?

The sequence diverges.

The sequence converges to 0.

The sequence converges to 1.

The sequence converges to [infinity].

Answers

Answer:

The sequence converges to 1.

Step-by-step explanation:

Sequence 2/4, 3/5, 4/6, 5/7

This sequence can be summarized as:

[tex]\sum_{n=0}^{\infty} \frac{n+2}{n+4}[/tex]

To test if it converges, we can calculate the limite of [tex]\frac{n+2}{n+4}[/tex] as n goes to infinite.

Limit:

[tex]\lim_{n \rightarrow \infty} \frac{n+2}{n+4}[/tex]

Considering only the terms with the highest exponent in the numerator and the denominator:

[tex]\lim_{n \rightarrow \infty} \frac{n+2}{n+4} = \lim_{n \rightarrow \infty} \frac{n}{n} = \lim_{n \rightarrow \infty} 1 = 1[/tex]

Thus, the sequences converges to 1.

25 is a perfect square. Which of the following is a perfect cube?
[A] 1
[B] 4
[C] 16
[D] 36

Answers

Answer:

the perfect cube is [A] 1

Answer: [A] 1

Explanation: 1x1x1 = 1

Perfect cubes are things what multiply by themselves 3 times!

The number of boysin a club is 48 and the number of girls is 20. Express this ratio in its simplest form

Answers

Answer:

the ratio is 12:5

Step-by-step explanation:

48:20 divided in half 24:10 divided in half 12:5

Combine these radicals. -6v100 + V36​

Answers

Answer:

-54

Step-by-step explanation:

[tex]-6\sqrt{100} +\sqrt{36}[/tex]

-6 * 10 + 6

-60 + 6

= -54

100 = 50 *2

       = 25 * 2 * 2

       = 5 * 5 * 2 * 2

       = [tex]5^{2} *2^{2}[/tex]

 5 * 2 = 10

~~~~~~~~~~

36 = 6 * 6 = 3 * 3 * 2 * 2 = 3^2 ^ 2^2

3*2 = 6

Which statement can be represented by the equation 18 divided by m = 7.2?

A number, m, less than 18 is seven point two.
Eighteen decreased by a number, m, is seven point two.
A number, m, divided by 18 is seven point two.
Eighteen divided by a number, m, is seven point two.

Answers

Answer:

A number,m,is seven point two . Eighteen divided by a number,m,is seven point two

Answer:

its c

Step-by-step explanation:

just to make it easier for you:)

Help with this question.

Answers

128³÷128²+272²+272³

128+272=400÷128*272=400

400÷400=1

It takes Priti 4 hours to drive from Ashdown to Bridgeton at an average speed of 50 mph. She then drives 30 miles from Bridgeton to Carton at an average speed of 40 mph. Assuming Priti doesn't stop, what is her average speed from Ashdown to Carton to 1 dp?

Answers

Answer:

Her average speed from Ashdown to Carton is approximately 48.4 mph

Step-by-step explanation:

The time and speed of the motion of Priti are as follows;

The time it takes Priti to drive from Ashdown to Bridgeton, t₁ = 4 hours

The average speed she drove from Ashdown to Bridgeton = 50 mph

The distance she drove from Bridgeton to Carton, d₂ = 30 miles

Her average speed from Bridgeton to Carton = 40 mph

The distance from Ashdown to Bridgeton, d₁ = 50 mph × 4 hours = 200 miles

The time it takes Priti to drive from Bridgeton to Carton, t₂ = 30 miles/(40 mph) = 0.75 hour

Average velocity, [tex]v_{ave}[/tex] = (Total distance traveled, Δd)/(Total time, Δt)

Δd = d₁ + d₂, Δt = t₁ + t₂

Δd = 200 miles + 30 miles = 230 miles

Δt = 4 hours + 0.75 hour = 4.75 hours

∴ The average velocity, [tex]v_{ave}[/tex] = (230 miles)/(4.75 hours)

∴ [tex]v_{ave}[/tex] = (920/19) mph

Her average speed from Ashdown to Carton, rouded to 1 d,p, [tex]v_{ave}[/tex] ≈ 48.4 mph

can someone help me find what a,b and c are?

Answers

Answer:

7 < 8.5 < 9

Step-by-step explanation:

Well < and > mean greater and less than so put it from the smallest to the largest

The polynomial function 1x) = 5x^5+16/5x-3
is graphed below
Which is a potential rational root of f(x) at point P?

Answers

First option. gboicboybkivnjonn

Which expression is equivalent to 3 sqrt32x8y10?

O 4x2y3(3 sqrt2x2y)
O 2x4y5(3 sqrt4)
O 2x2y3(3 sqrt4x2y)
O 4x4y5(3 sqrt2)​

Answers

Answer: [tex]2x^{2} y^{3} (\sqrt[3]{4x^{2} y} )[/tex]

Step-by-step explanation:

[tex]\sqrt[3]{32x^{8} y^{10} } =\sqrt[3]{2^{3} \cdot 2^{2} \cdot x^{2} \cdot (x^{2} )^{3} \cdot y \cdot (y^{3})^{3} } =2x^{2} y^{3} (\sqrt[3]{4x^{2} y} )[/tex]

Answer:

Option C :

               [tex]\sqrt[3]{32x^8y^10} = 2x^2 y^3 \sqrt[3]{4x^2 y}[/tex]

Step-by-step explanation:

[tex]\sqrt[3]{32x^8y^{10}}} = ( 32 x^8 y^{10})^{\frac{1}{3}}[/tex]

              [tex]= ( 2^5 \times x^8 \times y^{10})^{\frac{1}{3}}\\\\= ( 2^{5\times\frac{1}{3}} \times x^{8 \times \frac{1}{3}} \times y^{10 \times \frac{1}{3}})\\\\=(2^{\frac{3}{3} + \frac{2}{3}}} \times x^{\frac{6}{3} + \frac{2}{3}} \times y^{\frac{9}{3} + \frac{1}{3}})\\\\= 2 \times 2^{\frac{2}{3}} x^2 \times x^{\frac{2}{3}} \times y^3 \times y^\frac{1}{3}\\\\=2x^2 y^3 \times 2^{\frac{2}{3} \times }x^{\frac{2}{3}} \times y^{\frac{1}{3}}\\\\=2x^2 y^3 (2^2x^2 y)^{\frac{1}{3}}\\\\= 2x^2y^3 \ \sqrt[3]{ \ 4 x^2 \ y }[/tex]

25=125^k find the value of k​

Answers

Answer:

2/3

Step-by-step explanation:

25=125^k

5^2=5^3k

2=3k

k=2/3

The value of k is k = 2/3.

What is an exponent?

The number of times a number is multiplied by itself is referred to as an exponent.

Given that, [tex]25 = 125^k[/tex].

Follow the steps to find the value of k:

[tex]25 = 125^k\\\\5^2=(5^3)^k\\\\5^2=5^{3k}[/tex]

Since the base of the exponents is the same, it follows:

2 = 3k

k = 2/3

Hence, the value of k is k = 2/3.

Learn more about exponents here:

https://brainly.com/question/5497425

#SPJ2

3) Find the volume of the figure below. Round to the tenth place

Answers

Volume of cylinder= πr^2h

r=14.2/2=7.05

Quick note there are 2 cylinders one top and one buttom

V= πr^2h

= 2(π(7.05)^2x17.8)

Final answer = 5558.8 cm^3
Other Questions
Use identities to simplify Line l has a slope of the line through which of the following pair of points is perpendicular to l?A. (21,-4), (-5,7)B.(-7, -5),(4,7)c. (7-7), (-5,4)D.(-2,-5), (4,7) 8a+6ab+5b,-6a-ab-8b and - 4a+2ab+3bPls answer this question The product of two rational numbers is 2/5 , if one of them is -8/25, find the other.With steps explained. Which of the following BEST describes the policy of containment used during the ColdWar?a. Taking action to stop countries from invading the United Statesb. Taking action to stop people from leaving the United Statesc. Taking action to prevent the spread of communismd. Taking action to stop any new trade with China Ellen Co. has offered their customers a 1% discount off the amount owed if they pay within 15 days of receiving their bill. Handler Company owed Ellen Co. $2,185 as of May 1st and paid Ellen Co. on May 7th. How much cash did Handler Company send to Ellen Co. on May 7th? A family is planning to sell their home. If they want to be left with $116,100 after paying 10% of the selling price to a realtor as a commission, for how much must they sell the house? If 3 = 46, then 6 =look at image When a substance melts, evaporates or boils, which four of the following things are true for it's particles?Select four answers.A. Lose energyB. Gain energyC. Move moreD. Move lessE. Become further apartF. Become closer togetherG. Overcome the attractive forcesH. The forces between them hold them more in place Can somebody help me? I feel like this is wrong When the demand for a good is highly elastic with respect to price, how should a producer want to increase revenue? Why did Christopher Columbus believe that he landed in the West Indies when he reached the Americas? What event marks the beginning of the rising action in the most dangerous game A successful lease agreement is created so that both the lessee and lessor reap some benefits. Tax and depreciation write-offs are some critical reasons for leasing, but there are several other qualitative reasons for leasing. Below are two situations in which a firm must decide whether to lease or to buy a particular asset. Based on your understanding of the advantages to leasing from a qualitative perspective, what is the firm likely to do in each situationlease or buy? Assume all other quantitative factors remain constant.Compnay #1:Win Jet Corp. is a private-jet charter company. Due to increased demand during the summer, it needs to add three more jets to its fleet. Win Jet is more likely to ___________Compnay #2:Kiran owns a medium-sized printing business. She owns three one-color (black) printers and needs a color printer for volume print production. She wants to keep the operating expenses related to the color printer low, so she should ____________ a color printer. A) x = -2B) y =2C) y= -2 The kingdoms of north India could not defend themselves against the Turko-Afghan invasion. Write any two reasons. Doug bought a new car for $25,000. He estimates his car will depreciate, or lose value, at a rate of 20% per year. The value of his car is modeled by the equation V = P(1 r)t, where V is the value of the car, P is the price he paid, r is the annual rate of depreciation, and t is the number of years he has owned the car. According to the model, what will be the approximate value of his car after 4 and one-half years?$2,500$9,159$22,827$23,791 Explain the scholar definition of psychology which chemical formula describes carbon monoxide?a) COb) CO2c) C100d) CH4 what is the relationship between the energy of a wave and ruts frequency?